Warum ist die Hamiltonsche Mechanik wohldefiniert?

Beim erneuten Lesen des Formalismus der Hamiltonschen Mechanik bin ich auf ein Problem gestoßen, und es liegt in einer sehr einfachen Bemerkung.

Wenn ich mich nicht irre, führen wir in der Tat, wenn wir Mechanik unter Verwendung des Hamilton-Operators anstelle des Lagrange-Operators durchführen wollen, eine Legendre-Transformation am Lagrange-Operator durch, um den Hamilton-Operator zu erhalten. Dies wird im Fall eines 1-dimensionalen Problems wie folgt geschrieben:

H ( p , q ) = p q ˙ L ( q , q ˙ ) .
Beachten Sie, dass diese Transformation so ist H = H ( p , q ) , wo p = L q ˙ . Durch Variation von H , wir können es tatsächlich als Funktion von verifizieren p und q , sodass sie nun als die neuen unabhängigen Variablen des Problems betrachtet werden.

So weit, ist es gut. Es gibt jedoch ein Problem, und es liegt in der Tatsache, dass wir, damit diese Konstruktion gilt, die Lagrange-Funktion brauchen, um die Konvexität nicht zu ändern. Lassen Sie mich etwas formell schreiben, was ich über die Legendre-Transformation weiß:

Gegeben eine Funktion f : x f ( x ) , wir definieren die Funktion p : x p ( x ) durch die Relation d f d x = p . Angenommen d f d x invertierbar ist, können wir die Umkehrung von definieren p ( x ) , die wir nennen g : p g ( p ) . Dann die legendre Verwandlung von f ist f : p f ( p ) so dass d f d p = g ( p ) . Wir können auf vertrautere Weise schreiben, g ( p ) = x ( p ) da es das Gegenteil von ist p ( x ) .

Jedenfalls können wir das mit diesen Annahmen beweisen f = p g ( p ) f ( g ( p ) ) welches ist f = p x f ( x ) in "bekannter" Weise geschrieben. All dies soll nur darauf hinweisen, dass all diese Konstruktionen funktionieren und somit für die Existenz von f , wir brauchen die Bedingung dass f ( x ) andernfalls von konstanter Konvexität sein d f d x ist nicht umkehrbar und wir können nicht einmal definieren g ( p ) .

Wenn wir jedoch einen allgemeinen Lagrange betrachten, glaube ich nicht, dass dies immer der Fall ist. Einfach nehmen L = q ˙ 3 macht die Lagrange-Funktion nicht konstant konvex. Und doch verwenden wir immer den Hamilton-Operator, ohne jemals diese Konvexitätsbeschränkung zu überprüfen. Warum können wir das tun? Liegt es daran, dass wir uns für das lokale Verhalten unseres Lagrange interessieren? Aber selbst dann, was würden wir an einem Wendepunkt tun?

Oder liegt es daran, dass ein allgemeiner "physikalischer" Lagrange immer die Bedingung konstanter Konvexität erfüllt?

Was Sie getroffen haben, sind Phasenraumbeschränkungen. Es gibt eine Lösung für sie; siehe hier: physical.stackexchange.com/q/334005

Antworten (1)

Dies ist eine gute, aber ziemlich weit gefasste Frage. Lassen Sie uns die Positionsabhängigkeit unterdrücken q ich , ich { 1 , , n } , und explizite Zeitabhängigkeit t im Folgenden, um die Notation einfach zu halten.

Gegeben ein Lagrange L ( v ) , können mehrere Dinge schief gehen, wenn wir versuchen, eine Legendre-Transformation durchzuführen, um einen Hamilton-Operator zu konstruieren H ( p ) . Definieren Sie zur späteren Bequemlichkeit die Funktion

(0) h ( v , p )   :=   p j v j L ( v ) .
Wie auf der Wikipedia-Seite erklärt, gibt es mindestens zwei Definitionen der Legendre-Transformation :

(1) H ( p )   :=   sup v h ( v , p ) ,

und

(2) g j ( v )   :=   L ( v ) v j , f   :=   g 1 , H ( p )   :=   h ( f ( p ) , p )   =   p ich f ich ( p ) L f ( p ) .

Die frühere Definition 1 (die sogenannte Legendre-Fenchel-Transformation) funktioniert tendenziell besser für konvexe (möglicherweise nicht differenzierbare) Lagrange-Operatoren L , während die letztere Definition 2 (die historisch die ursprüngliche Legendre-Transformation ist) tendenziell besser für differenzierbare (möglicherweise nicht-konvexe) Lagrange-Funktionalitäten funktioniert L . Aber generell müssen wir mehr Auflagen machen, wie weiter unten deutlich wird. (Es versteht sich von selbst, dass diese Bedingungen in tatsächlichen Systemen verletzt werden können.)

In Bezug auf die frühere Definition 1 siehe auch diesen , diesen Phys.SE-Beitrag und diesen Math.SE-Beitrag. Arbeiten in der strengen mathematischen / statistischen Physik scheinen Definition 1 zu bevorzugen. In der klassischen Mechanik und Feldtheorie verwenden wir traditionell die letztere Definition 2, auf die wir uns von nun an in dieser Antwort konzentrieren werden.

Aus technischen Gründen stellt man oft eine Regelmäßigkeitsbedingung, dass die n × n Hessische Matrix 1

(3) H ich j   :=   g j ( v ) v ich   =   2 L ( v ) v ich v j
hat einen konstanten Rang r .

Wenn der Rang r = n maximal ist, garantiert der Umkehrfunktionssatz die lokale (aber nicht die globale) Existenz einer Umkehrfunktion f .

Wenn eine global definierte Umkehrfunktion f existiert, dann die Konstruktion des Hamiltonoperators H ( p ) wurde in diesem Phys.SE-Beitrag besprochen.

Wenn der Rang r < n nicht maximal ist, dann ist die Legendre-Transformation singulär. Dann können wir nicht alle Geschwindigkeiten ausdrücken v ich als Funktionen von Impulsen p j , nur r davon. Dies führt zu n r primäre Einschränkungen . Das stellt sich in diesem Einzelfall förmlich heraus r < n , ist es immer noch möglich , einen Hamilton - Operator in einem erweiterten Phasenraum lokal so zu konstruieren , dass die Legendre - Transformation eine Involution ist . Siehe zB meine Phys.SE-Antwort hier und die darin enthaltenen Referenzen.

Abschließend sei noch erwähnt, dass wir die Legendre-Transformation auch mathematisch definieren können H ( p ) , gibt es keine Garantie, dass es physikalisch sinnvoll ist. In der QM benötigen wir zB normalerweise den Hamilton-Operator H ^ ist selbstadjungiert und nach unten beschränkt.

Verweise:

  1. Hugo Touchette, Legendre-Fenchel verwandelt sich auf den Punkt , 2005.

--

1 Übrigens OPs Beispiel

(4) L ( v )   =   1 3 v 3
verletzt die konstante Rangbedingung (3). Für dieses Beispiel ergibt sich Definition 1
(5) H ( p )   =   ,
während Definition 2 einen zweiwertigen Hamilton-Operator ergibt
(6) g ( v )   =   v 2 , f ( p )   =   ± p , H ( p )   =   ± 2 3 p 3 2 , p     0.
Es scheint, dass man den negativen Zweig verwerfen sollte, um ein brauchbares physikalisches Modell zu haben.